Identify the slope type of the graph shown above.
Question 15 options:

A)

Undefined

B)

Zero

C)

Positive

D)

Negative

Identify The Slope Type Of The Graph Shown Above.Question 15 Options:A) UndefinedB) ZeroC) PositiveD)

Answers

Answer 1
Answer is B. Zero
Reason
Pick any points on that line, they will have the same y value , it means they lie on a horizontal line. The slope of such a line is 0.
Answer 2

Answer:

Zero

Step-by-step explanation:


Related Questions

Question is in the picture

Question is in the picture

Answers

Answer:

(a)  D = 72.05 dB  (2 d.p.)

(b)  I = 2.51 W/m²  (2 d.p.)

Step-by-step explanation:

Given function:

\(\boxed{D(I)=10 \log_{10}(I)+120, \quad I\geq 0}\)

where:

D is the sound intensity level (in dB).I is the sound intensity (in W/m²).Part (a)

Given:

\(I=1.6 \times 10^{-5}\; \sf W/m^2\)

Substitute the given value of I into the given function and solve for D:

\(\begin{aligned}\implies D(1.6 \times 10^{-5}) & =10 \log_{10}(1.6 \times 10^{-5})+120\\& = 10 \log_{10}(0.000016)+120\\& = 10 (-4.795880017)+120\\& = -47.95880017+120\\&=72.04119983\\&=72.04\; \sf dB\;(2\:d.p.)\end{aligned}\)

Part (b)

Given:

D = 124 dB

Substitute the given value of D into the given function and solve for I:

\(\begin{aligned}\implies 124 & =10 \log_{10}(I)+120\\4 & =10 \log_{10}(I)\\10 \log_{10}(I)&=4\\\log_{10}(I)&=0.4\\10^{\log_{10}(I)}&=10^{0.4\\I&=2.511886432\\I&=2.51\; \sf W/m^2\; (2\:d.p.)\end{aligned}\)

A valid study Group of answer choices uses random sampling. results in a causal relationship between the independent and dependent variables. portrays the population being studied. measures what it is supposed to. verifies expected results.

Answers

A valid study is one that portrays the population being studied and measures what it is supposed to. It must use random sampling and produce expected results to be deemed a reliable source. However, such studies do not necessarily result in a causal relationship between the independent and dependent variables.

The primary goal of any study is to depict the population being investigated in an accurate and unbiased manner. This involves selecting an appropriate sample of the population to study. This is why a study must use random sampling, which means every member of the population has an equal chance of being selected. This helps avoid sample bias, which is when a sample is chosen in a way that it is not representative of the population, leading to incorrect or inaccurate conclusions.

A valid study must measure what it is supposed to, which means that the study should be designed to measure what it claims to measure. For instance, a study on the effects of sleep on memory should use measures that accurately assess memory and sleep. If the study is designed to assess memory but fails to do so, the results will be unreliable. Finally, a valid study verifies expected results.

This means that the study should produce results that are consistent with what is expected based on prior research or theory. However, this does not mean that a study necessarily results in a causal relationship between the independent and dependent variables. Rather, such a relationship can only be inferred if the study is well-designed and the results are properly analyzed and interpreted.

To know more about independent visit:

https://brainly.com/question/27765350

#SPJ11

Evaluate the following expression.
32÷23+12

Answers

Answer:

32÷23+12

1.39+12

13.39

QUESTION 2 How many arrangements of the letters in FULFILLED have the following properties simultaneously? - No consecutive F′s. - The vowels E,I,U are in alphabetical order. - The three L′s are next to each other.

Answers

There are 4 arrangements of the letters in FULFILLED that satisfy all the given properties simultaneously.

To determine the number of arrangements, we can break down the problem into smaller steps:

⇒ Arrange the three L's together.

We treat the three L's as a single entity and arrange them among themselves. There is only one way to arrange them: LLL.

⇒ Arrange the remaining letters.

We have the letters F, U, F, I, E, D. Among these, we need to ensure that no two F's are consecutive, and the vowels E, I, and U are in alphabetical order.

To satisfy the condition of no consecutive F's, we can use the concept of permutations with restrictions. We have four distinct letters: U, F, I, and E. We can arrange these letters in a line, leaving spaces for the F's. The number of arrangements can be calculated as:

P^UFI^E = 4! / (2! * 1!) = 12,

where P represents permutations.

Next, we need to ensure that the vowels E, I, and U are in alphabetical order. Since there are three vowels, they can be arranged in only one way: EIU.

Multiplying the number of arrangements from Step 1 (1) with the number of arrangements from Step 2 (12) and the number of arrangements for the vowels (1), we get:

Total arrangements = 1 * 12 * 1 = 12.

Therefore, there are 4 arrangements of the letters in FULFILLED that satisfy all the given properties simultaneously.

To know more about permutations with restrictions, refer here:

https://brainly.com/question/33193507#

#SPJ11

Dr. Shaw wants to conduct a large-scale study examining the attitudes that 20,000 adolescents hold toward their high school education. He starts by doing a small version of the study, using only 40 students to make sure his measures have adequate reliability and validity. This smaller study is an example of

Answers

The smaller study conducted by Dr. Shaw using only 40 students to test the reliability and validity of his measures is an example of a pilot study or a feasibility study.

A pilot study is a small-scale preliminary study that is conducted before a larger-scale study to evaluate the feasibility of the research design, data collection methods, and measurement instruments.

It helps researchers identify any potential issues or challenges that may arise in the larger study and allows them to make necessary adjustments or improvements.

In this case, Dr. Shaw is using the pilot study to assess the reliability and validity of his measures in measuring the attitudes of adolescents toward their high school education.

By selecting a smaller sample size, he can gather data on a manageable scale and evaluate the consistency and accuracy of the measures before implementing them in the larger-scale study involving 20,000 adolescents. The pilot study helps ensure that the measures are reliable and valid, increasing the confidence in the results of the subsequent larger study.

Learn more about reliability here : brainly.com/question/29886942

#SPJ11

What is 6.16 rounded to the nearest tenth?

Answers

Answer:

6.2

Step-by-step explanation:

Okay so in order to solve this equation you are going to have to round up the tenths place which is the space after the decimal point.

6.16 = 6.2

because 6 is above 5 you are able to round up to ten turning 6.16 to 6.2

Hope this helps you!!!


Student Council sells soft drinks at basketball games and makes $ 1.50 from each. If they pay $ 75 to rent the concession stand, how many soft drinks would they have to sell to make $ 250 profit?
F. 116
G. 117
H. 167
J. 217

Answers

Answer: 166 Soft Drinks

Step-by-step explanation:

250 ÷ 1.50 = 166

What does 100% increase mean???

Answers

Answer:

double whatever you got

100 percent increase of 2 is 4

Step-by-step explanation:

hopes this helps please mark brainliest

Answer:

It means that whatever you have just doubled.

Step-by-step explanation:

If I have a total of 100 widgets, that is 100%.  If I have a 100% increase of that value, that means I now have 200 widgets.

What is the range of this data set?
Length in Roses length in cm 22cm 23cm 24cm 25cm 26cm Number of roses 2 4 5 3 1

What is the range of this data set?Length in Roses length in cm 22cm 23cm 24cm 25cm 26cm Number of roses

Answers

The range of a data set is the difference between the maximum and minimum values.

In this case, the minimum value is 22 cm (the shortest rose length) and the maximum value is 26 cm (the longest rose length).

Therefore, the range of the data set is:

Maximum value - Minimum value = 26 cm - 22 cm = 4 cm

So, the range of the data set is 4 cm.

The Nieto’s family bought a house several years ago for $120,000 this year they sold it for 150,000 find the percent of inscrease

Answers

Answer:

25% increase

Step-by-step explanation:

Percent Increase= new value-old value divided by the old value

New Value= 150,000

Old Value= 120,000

150,000-120,000 divided by 120,000

0.25=25%

Answer:

25%

Step-by-step explanation:

1) They bought the house for $120,000 and sold it for $150,000 whcich is simply 150,000/120000 simplified is 5/4.

2) In order to get it as a percentage you multiply the 5/4 by 100 getting 125. To work out the percentage increase you do 125-100 = 25% making the percentage increase 25%

Thanks for reading,if I've missed anything out please let me know!

Declaring variables - Declare two integer variables x and y, - Assign them any values. - Print addition/subtraction/multiplication and division of these two variables on to the screen
Submission Task (- Grade 1%) Follow the same steps asin Exercise 2, but change the step 2 to ask the user for input forthese values by using Scanner class.

Answers

Two integer variables x and y, prompts the user to enter values for them using the Scanner class, and performs addition, subtraction, multiplication, and division operations on those variables:

import java.util.Scanner;

public class VariableOperations {

   public static void main(String[] args) {

       Scanner scanner = new Scanner(System.in);

       System.out.print("Enter the value for x: ");

       int x = scanner.nextInt();

      System.out.print("Enter the value for y: ");

       int y = scanner.nextInt();

       // Addition

       int addition = x + y;

       System.out.println("Addition: " + addition);

       // Subtraction

       int subtraction = x - y;

       System.out.println("Subtraction: " + subtraction);

       // Multiplication

       int multiplication = x * y;

       System.out.println("Multiplication: " + multiplication);

       // Division

       if (y != 0) {

           double division = (double) x / y;

           System.out.println("Division: " + division);

       } else {

           System.out.println("Cannot divide by zero.");

       }

   }

}

This code prompts the user to enter values for x and y, performs the four basic arithmetic operations, and displays the results on the screen.

Learn more about variables here:

https://brainly.com/question/29696241

#SPJ11

Find the sine of ∠D.

Write your answer as an integer or as a decimal rounded to the nearest hundredth.

Find the sine of D.Write your answer as an integer or as a decimal rounded to the nearest hundredth.

Answers

We know

\(\\ \sf{:}\Rrightarrow sin\Theta=\dfrac{Perpendicular}{Hypotenuse}\)

Now

\(\\ \sf{:}\Rrightarrow sinD=\dfrac{FE}{DE}\)

\(\\ \sf{:}\Rrightarrow sinD=\dfrac{55}{73}\)

Prove AB is congruent to BC given BE bisects DBC and BE is parallel to AC

Prove AB is congruent to BC given BE bisects DBC and BE is parallel to AC

Answers

AB is congruent to BC given BE bisects DBC and BE is parallel to AC is proved .

What is congruent ?

Congruent refers to having the same shape and size. In mathematics, two objects are said to be congruent if they are identical in shape and size, and can be superimposed onto one another. The symbol used to represent congruence is ≅. Congruence applies to various geometric objects, such as triangles, rectangles, circles, and more. When two objects are congruent, they have all corresponding angles equal and all corresponding sides equal in length.

Step 1: Statement: \($\angle DBE = \angle EBC$\)

Reason: Given that overline BE bisects \($\angle DBC$\)

Step 2: Statement: \($\angle DBC + \angle EBC = 180^\circ$\)

Reason: Angle sum property of a straight line.

Step 3: Statement: \($\angle ABC + \angle EBC = 180^\circ$\)

Reason: Angles on a straight line sum to \(180^\circ$, and $\overline{BE} || \overline{AC}$\) implies that \(\angle ABC$ and $\angle EBC$\) are co-interior angles.

Step 4: Statement: \($\angle ABC = \angle DBC$\)

Reason: From step 2 and step 3, \($\angle ABC + \angle EBC = \angle DBC + \angle EBC = 180^\circ$\). Thus, \($\angle ABC = \angle DBC$\).

Step 5: Statement: \($\triangle ABE \cong \triangle CBE$\)

Reason: By the angle-angle-side congruence criterion, since \($\angle DBE = \angle EBC$\) (from step 1) and \($\angle ABC = \angle DBC$\) (from step 4), and \($\overline{BE}$\) is common to both triangles.

Step 6: Statement: \($AB = BC$\)

Reason: By step 5, \($\triangle ABE \cong \triangle CBE$\), so corresponding sides are congruent, including \($\overline{AB} \cong \overline{BC}$\).

Therefore, AB is congruent to BC given BE bisects DBC and BE is parallel to AC is proved .

To know more about congruent visit :

https://brainly.com/question/11949261

#SPJ1

The Ski Club sells 180 keychains. A third of the keychains sold are blue and 1/6 of the remaining keychains are white. The rest of the keychains are red. How many keychains are red?

Answers

Answer:

Step-by-step explanation:

blue = ⅓×180 = 60

180-60 = 120 are not blue.

white = ⅙×120 = 20

120-20 = 100 are red

Find the volume of the figure. Do NOT include units.

Find the volume of the figure. Do NOT include units.

Answers

Step-by-step explanation:

hope this can help you with your work, you can clarify or point out any mistakes that I make or any steps that you do not understand

Find the volume of the figure. Do NOT include units.

HELPPPPPPPPPP :(

If there are two doors, each 36 inches wide.
Door A opened to 86° angle. Door B opened to 82°

which door's outer edge is farther from its closed position? and please explain it ♡​

Answers

Answer:

B its B

Step-by-step explanation:

The HIV incidence for a particular community is 1.0 per month. Assume that the number of new HIV infections follow a Poisson distribution. Find the probability that, in a 5 month period, there will at least two new HIV infections (i.e. two or more). (In the tables provided we use u for the population mean number of events; in your lectures the lecturer used X. Either is acceptable).
(HINT 1: if u is given for 1 time interval as = x; and you are now observing 5 time intervals, p=5"x.)
(HINT 2: Since, in theory, Poisson events may rise to infinity, best way to determine p for "more than" problems, such as ">X", is to determine p for sX; and then subtract this from 1.)
0.0404
0.0174
0.9596
0.8753

Answers

The probability that there will be at least two new HIV infections in a 5 month period is 0.9596. Therefore, the correct option is (C) 0.9596.

The number of new HIV infections in a 5 month period follows a Poisson distribution with mean (u) equal to λ = 5 x 1 = 5, since the incidence rate is given for one month.

Let X be the number of new HIV infections in a 5 month period. Then,

P(X ≥ 2) = 1 - P(X < 2)

To calculate P(X < 2), we can use the Poisson probability formula:

P(X = k) = e^(-λ) * (λ^k) / k!

where k is the number of new HIV infections in a 5 month period.

So,

P(X < 2) = P(X = 0) + P(X = 1)

= e^(-5) * (5^0) / 0! + e^(-5) * (5^1) / 1!

= 0.0067 + 0.0337

= 0.0404

Therefore,

P(X ≥ 2) = 1 - P(X < 2)

= 1 - 0.0404

= 0.9596

Hence, the probability that there will be at least two new HIV infections in a 5 month period is 0.9596. Therefore, the correct option is (C) 0.9596.

Learn more about  probability  from

https://brainly.com/question/30390037

#SPJ11

Penelope invested $840 in an account paying an interest rate of 1 3 8 1 8 3 ​ % compounded continuously. Ella invested $840 in an account paying an interest rate of 1 1 4 1 4 1 ​ % compounded daily. To the nearest hundredth of a year, how much longer would it take for Ella's money to double than for Penelope's money to double?

Answers

Using continuous compounding and compound interest, it is found that it takes 5.04 years longer for Ella's money to double than for Penelope's money to double.

The amount of money, after t years, using continuous compounding, is given by:

\(A(t) = A(0)e^{rt}\)

In which:

A(0) is the initial amount.r is the interest rate, as a decimal.

In this problem, Penelope's interest rate, as a percentage, is:

\(1\frac{3}{8}\% = 1 + \frac{3}{8} = 1.375\%\)

Hence \(r = 0.01375\)

The time to double is t for which A(t) = 2A(0), then:

\(A(t) = A(0)e^{rt}\)

\(2A(0) = A(0)e^{0.01375t}\)

\(e^{0.01375t} = 2\)

\(\ln{e^{0.01375t}} = \ln{2}\)

\(0.01375t = \ln{2}\)

\(t = \frac{\ln{2}}{0.01375}\)

\(t = 50.41\)

It takes 50.41 years for Penelope's amount to double.

Compound interest:

\(A(t) = P\left(1 + \frac{r}{n}\right)^{nt}\)

A(t) is the amount of money after t years.  P is the principal(the initial sum of money).  r is the interest rate(as a decimal value).  n is the number of times that interest is compounded per year.  t is the time in years for which the money is invested or borrowed.

For Ella's, we have that:

Compounded daily, hence \(n = 365\).Rate, as a percent, of \(1\frac{1}{4} = 1 + \frac{1}{4} = 1.25\%\), hence \(r = 0.0125\)

The time to double is t for which A(t) = 2P, hence:

\(A(t) = P\left(1 + \frac{r}{n}\right)^{nt}\)

\(2P = P\left(1 + \frac{0.0125}{365}\right)^{365t}\)

\((1.00003424658)^{365t} = 2\)

\(\log{(1.00003424658)^{365t}} = \log{2}\)

\(365t\log{1.00003424658} = \log{2}\)

\(t = \frac{\log{2}}{365\log{1.00003424658}}\)

\(t = 55.45\)

It takes 55.45 years for Ella's amount to double.

55.45 - 50.41 = 5.04

It takes 5.04 years longer for Ella's money to double than for Penelope's money to double.

A similar problem is given at https://brainly.com/question/24507395

Which of the following is/are correct? (Select all that apply. You have only one submission for this question.) If u and v are parallel, then either u.v=jul lvl or u. V-lul lvl. The vector projuv is parallel to v If u and v are orthogonal, then u xv = 0. The expression u. (vw) is meaningful and the result is a scalar. Suppose u: O. If u xv=uxw, it follows that vw. If u and v are parallel, then u xv = 0. The expression u u can be negative

Answers

The correct statements are If u and v are orthogonal, then u × v = 0. The vector proj_v u is parallel to v. If u and v are parallel, then u × v = 0.

The statement "If u and v are parallel, then either u · v = |u||v|" is incorrect. The correct statement is "If u and v are parallel, then u · v = |u||v|".

The statement "The expression u.(vw) is meaningful and the result is a scalar" is incorrect. The expression u.(vw) is not meaningful because the dot product is only defined for vectors of the same dimension.

The statement "Suppose u: O. If u × v = u × w, it follows that v = w" is incorrect. The correct statement is "Suppose u ≠ 0. If u × v = u × w, it follows that v - w is parallel to u".

Finally, the statement "The expression u · u can be negative" is incorrect. The dot product u · u is always non-negative, and is only equal to zero if and only if u = 0

Learn more about orthogonal here

https://brainly.com/question/31043750

#SPJ11.

Superstar Toy Shop is having its annual holiday sale, when every toy in the store gets marked down. During the sale, Pamela purchases 4 Mighty Mare toy ponies to add to her collection, each at $3 less than its full price. Pamela pays a total of $52.
Which equation can you use to find the amount of money, x, each Mighty Mare toy pony costs at full price?

Answers

Let x be the full price of each Mighty Mare toy pony.

During the holiday sale, Pamela purchased each toy at $3 less than its full price. Therefore, the price she paid during the sale was:

x - $3

Since she bought 4 of these toys, her total cost during the sale was:

4(x - $3)

We also know that Pamela paid a total of $52 during the sale. Therefore, we can set up an equation:

4(x - $3) = $52

Simplifying this equation, we get:

4x - $12 = $52

Adding $12 to both sides, we get:

4x = $64

Dividing both sides by 4, we get:

x = $16

Therefore, each Mighty Mare toy pony costs $16 at full price.

Let's assume that the full price of each Mighty Mare toy pony is x dollars. Since Pamela purchased 4 Mighty Mare toy ponies, each at $3 less than its full price, the cost of each toy pony during the sale would be (x - 3) dollars.

To find the equation that represents this situation, we can set up the equation based on the given information:

4 * (x - 3) = 52

In this equation, 4 represents the number of Mighty Mare toy ponies purchased, (x - 3) represents the cost of each toy pony during the sale, and 52 represents the total amount paid by Pamela.

By solving this equation, we can determine the value of x, which represents the full price of each Mighty Mare toy pony.

a decimal number lies between 0.7 and 0.8 . it has three digits to the right of the decimal point. thus, it is of the following format: it is the largest number between 0.7 and 0.8 .the sum of the digits of this decimal number is 24 . find the decimal number.

Answers

The decimal number that lies between 0.7 and 0.8, has three digits to the right of the decimal point, and whose sum of the digits is 24 is 7.98.

A decimal number is a number that has a fractional part represented by a decimal point. The digits to the right of the decimal point represent values that are smaller than 1.

In this problem, we are given that the decimal number has three digits to the right of the decimal point and that its sum of the digits is 24. Let's call the decimal number x. We can write an equation to represent the sum of the digits:

x = a + b/10 + c/100

Where a, b, and c are the digits of x.

We know that 0.7 < x < 0.8 and that the sum of the digits of x is 24, so we can write two more equations:

0.7 < a + b/10 + c/100 < 0.8

a + b + c = 24

By solving these three equations, we can find the decimal number x.

We know that a must be 7 because 0.7 < x < 0.8, so a = 7. We also know that b + c = 24 - 7 = 17. Now we need to find two digits b and c such that b + c = 17 and b/10 + c/100 is between 0 and 0.1.

The largest value of b that still satisfies this condition is 9. This means that

=> c = 17 - 9 = 8.

Therefore, the decimal number

=> x = 7 + 9/10 + 8/100 = 7.98.

To know more about decimal here.

https://brainly.com/question/9543292

#SPJ4

what is the price of a 3.08 nnual coupon bond with a face value of $1,000 and a maturity of 10 years? the market’s ytm on a comparable risk and term bond is 7.26hegg

Answers

The price of a 3.08% annual coupon bond with a face value of $1,000 and a maturity of 10 years, given a market yield to maturity (YTM) of 7.26%, can be calculated using the present value formula and the bond's cash flows.

To calculate the price of the bond, we need to discount the bond's cash flows to their present values. The bond pays an annual coupon of 3.08% of the face value, which is $1,000. This means that the bondholder will receive $30.80 in coupon payments every year for 10 years. At maturity, the bondholder will also receive the face value of $1,000.

To determine the present value of the bond's cash flows, we discount each cash flow by the market yield to maturity (YTM) of 7.26% per year. This YTM represents the market's required rate of return for bonds with similar risk and maturity.

Using financial calculations or a spreadsheet, we can calculate the present value of the bond's cash flows and sum them up to find the bond's price. The formula for calculating the price of a bond is:

Price = (Coupon payment / (1 + YTM)¹) + (Coupon payment / (1 + YTM)²) + ... + (Coupon payment / (1 + YTM)ⁿ) + (Face value / (1 + YTM)ⁿ)

In this case, the bond has a maturity of 10 years, so we discount the coupon payments and face value for each year using the YTM of 7.26%. After calculating the present value for each cash flow, we sum them up to find the bond's price.

Learn more about Coupon

brainly.com/question/32114615

#SPJ11

Mr. Mole left his burrow that lies below the ground and started digging his way at a constant rate deeper into the ground. Let y represent Mr. Mole's altitude (in meters) relative to the ground after x minutes. Which of the following could be the graph of the relationship?

Answers

Answer:

We know that:

The initial conditions are -7 meters at 0 minutes.

Then, after 6 minutes, he was 16 meters below the ground.

According to these two simple facts we can found the linear function that describes this problem. First, the problem says that Mr. Mole is descending at a constant rate, which is the slope of the function. Now, to calculate the slope we need to points, which are  and , where t-values are minutes, and y-values are meters. You can see, that the first point is the initial condition and the second point is 6 minutes later.

So, we calculate the slope:

From the slope we can see that Mr. Mole is descending, because it has a negative sign. Also, the point  is on the y-axis, because t is null, so -7 is part of the function. Therefore the function that describes this problem is:

Step-by-step explanation:

please help me answer will get brainliest

please help me answer will get brainliest

Answers

Answer:

32% is 0.32

6% is 0.06

93% is 0.93

150% is 1.5

12.5% is 0.125

Hope this helps!

If f(x) = 2(x − 5), find f(8). 4 6 8 16

Answers

Answer:

f(8) = 6

Step-by-step explanation:

Plug in 8 as x in the function:

f(x) = 2(x − 5)

f(8) = 2(8 - 5)

f(8) = 2(3)

f(8) = 6

Answer:

\(\large \boxed{\mathrm{6}}\)

Step-by-step explanation:

\(f(x)=2(x-5)\)

Replace x with 8 to find f(8).

\(f(8)=2(8-5)\)

Evaluate.

\(f(8)=2(3)\)

\(f(8)=6\)

a cardboard box without a lid is to have a volume of 23,328 cm3. find the dimensions that minimize the amount of cardboard used. (let x, y, and z be the dimensions of the cardboard box.) (x, y, z) =

Answers

The dimensions (x, y, z) that minimize the amount of cardboard used for a box with a volume of 23,328 cm³ are (28, 28, 30).

1. Given the volume, V = x*y*z = 23,328 cm³.


2. The surface area, which represents the amount of cardboard used, is S = x*y + x*z + y*z.


3. To minimize S, we need to use calculus. First, express z in terms of x and y using the volume equation: z = 23,328 / (x*y).


4. Substitute z into the surface area equation: S = x*y + x*(23,328 / (x*y)) + y*(23,328 / (x*y)).


5. Now find the partial derivatives dS/dx and dS/dy, and set them equal to zero.


6. Solve the system of equations to get x = 28 and y = 28.


7. Plug x and y back into the equation for z: z = 23,328 / (28 * 28) = 30.


So the dimensions that minimize the amount of cardboard used are (28, 28, 30).

To know more about partial derivatives click on below link:

https://brainly.com/question/31397807#

#SPJ11

g(x)=4x-8 find g(7) i really need help

Answers

Just replace the variable "x" with "7":

So we have 4(7) - 8 or 28 - 8 which is 20.

So g(x) = 20.

Answer:

g(7) = 20

Step-by-step explanation:

To evaluate g(7), substitute x = 7 into g(x), that is

g(7) = 4(7) - 8 = 28 - 8 = 20

A square pyramid has a base with an area of 20 square meters, and its lateral faces have a slant height of x meters. Sydney is constructing a second square pyramid with the same size base, but the lateral faces of her pyramid have a slant height twice as long, 2x. Which statement best describes how the surface area of Sydney’s pyramid compares to the surface area of the original pyramid?
Sydney’s pyramid will have the same surface area because the One-half in the expression for the area of the triangular faces will make up for the slant height being doubled.
Sydney’s pyramid will have the same surface area because the slant height is not used when finding surface area.
Sydney’s pyramid will have a surface area that is exactly double the original pyramid’s because the slant height is used when finding the area of every lateral face.
Sydney’s pyramid will have a surface area that is greater than the original pyramid’s but not double the area because the slant height is not used when finding the area of the base.

Answers

1. Consider the pyramid ABCDE in the figure 1 attached:

Each side of the square base equals  (units)

Area of 1 lateral side of pyramid ABCDE is

so the total lateral area is  (units squared)

and the total surface area is  (units squared)

2. Now consider the second figure KLMNP, Sydney's pyramid:

The lateral area is

so the total area of the second pyramid is

So 2 things can be said about the areas:

i) the lateral area of Sydney's pyramid is times larger than the lateral area of the original pyramid.

ii) The total area of Sydney's pyramid is units squared larger than the total area of the original pyramid.

A square pyramid has a base with an area of 20 square meters, and its lateral faces have a slant height
A square pyramid has a base with an area of 20 square meters, and its lateral faces have a slant height

Calculating brilliance in epidemiology Context. What follows is a data table showing the development of brilliance among a small class of PHE 450 students. NOTE: Student #8 came in as an existing case of brilliance and did not develop brilliance as a result of exposure to PHE 450. Student WK 1 WK 2 WK 3 WK 4 WK 5 WK6 WK 7 WK 8 WK 9 WK 10 CASE CASE CASE CASE DROP 1 2 3 4 5 6 7 8 9 10 11 12 CASE CASE CASE DROP CASE DROP ASSIGNMENT Referring to the data above, please answer the following questions What is the point prevalence of brilliance at the end of Week 1? What is the point prevalence of brilliance at the end of Week 2? • What is the point prevalence of brilliance at the end of Week 3? • Using person-weeks as your denominator, what is the incidence of brilliance over the course of the 10-week course?

Answers

The point prevalence of brilliance at the end of Week 1 is 0.08 or 8%.

The point prevalence of brilliance at the end of Week 2 is 0.17 or 17%.

The point prevalence of brilliance at the end of Week 3 is 0.33 or 33%.

Using person-weeks as denominator, the incidence of brilliance over the course of the 10-week course is 0.017 or 1.7%

In epidemiology context, brilliance can be calculated through calculating point prevalence, cumulative incidence, and incidence rate. The provided data table can be used to determine the point prevalence, incidence, and incidence rate of brilliance among PHE 450 students. So, the calculations of point prevalence, cumulative incidence, and incidence rate based on the provided data are as follows:

The point prevalence of brilliance at the end of Week 1 can be calculated by the following formula; Point prevalence = Total number of existing cases at a given time ÷ Total population at that time

Student #8 was the only existing case of brilliance at the beginning of Week 1, so the point prevalence of brilliance at the end of Week 1 is; Point prevalence = 1 ÷ 12 = 0.08 or 8%.

The point prevalence of brilliance at the end of Week 2 can be calculated by the following formula; Point prevalence = Total number of existing cases at a given time ÷ Total population at that time

Student #3 and Student #8 were existing cases of brilliance at the beginning of Week 2, so the point prevalence of brilliance at the end of Week 2 is; Point prevalence = 2 ÷ 12 = 0.17 or 17%.

The point prevalence of brilliance at the end of Week 3 can be calculated by the following formula; Point prevalence = Total number of existing cases at a given time ÷ Total population at that time

Student #3, #4, #6, and #8 were existing cases of brilliance at the beginning of Week 3, so the point prevalence of brilliance at the end of Week 3 is; Point prevalence = 4 ÷ 12 = 0.33 or 33%.

The incidence of brilliance can be calculated by the following formula; Incidence = Total number of new cases ÷ Total person-weeks of observation

Student #5 and Student #7 developed brilliance during the 10-week course, so the incidence of brilliance over the course of the 10-week course is; Incidence = 2 ÷ 120 = 0.017 or 1.7%.

To know more about denominator refer here:

https://brainly.com/question/931030#

#SPJ11

The 40-hour work week did not become a U.S. standard until 1940. Today, many white-collar employees work more than 40 hours per week because management demands longer hours or offers large monetary incentives. A random sample of 26 white-collar employees worked on average 42.08 hours per week. Is there any evidence that the true mean number of hours worked per week by white-collar employees is greater than 40? Assume that the population standard deviation is 3.39 hours. Please use the exact value (from R) for all critical values. c) Is there any evidence to suggest the true mean number of hours worked per week by white-collar workers is greater than 40? Perform the hypothesis test at a 0.5% significance level. (0.5 pts.) Calculate the test statistic. 3.13 You are correct. Your receipt no. is 152-5185 Previous Tries (0.5 pts.) Calculate the p-value. Please write your answer in scientific notation using an E for the exponent and including at least 3 decimal places. For example, 1.234 x 10-5 would be written as 1.234E-5. 8.782x10-4 (0.5 pts.) d) Calculate the appropriate 99.5% bound that is consistent with what you did in parts b) and c).

Answers

The necessary assumptions are to get the sample for random sampling and ensure they are normal.

a. The required assumptions to form hypothesis are:Sample has to be drawn from random sampling.Sample has to be approximately normalThe samples have to be independent

b. The standard deviation is known hence we have to make use of the z distribution.

c. test statistic

42.08 - 40 / (3.39 /√26)

= 2.08 / 0.6648

= 3.129

≈3.13

p value

p(Z > 3.13)

= 0.0008770

8.7e-4

d. Z critical value = 2.81

42.08 +- 2.81 * 3.39/√26

= 42.08 +- 2.81*0.6648

=  42.08 - 1.868, 42.08 + 1.868

= 40.212  ,43.948

Read more on statistics here:

https://brainly.com/question/19243813

#SPJ1

Other Questions
How did the Persian Wars lead to the Peloponnesian War? How many moles of magnesium chloride are produced when 4.39 moles of magnesium are placed in a beakercontaining only 250.0 grams of hydrochloric acid? 3/10 of a bag oh jelly were red, and 2/10 of the bag were orange. What fraction of the bag was red and orange ? why does the light bulb in a circuit turn on when the switch is closed? responses the switch changes the circuit from series to parallel. the switch changes the circuit from series to parallel. the switch changes the direction of the flow of electrons. the switch changes the direction of the flow of electrons. the switch absorbs the electrical energy. the switch absorbs the electrical energy. closing the switch completes the circuit, making it a closed circuit. closing the switch completes the circuit, making it a closed circuit. If the releases 39.4 kj of energy, how many kilocalories does it release? (1 cal = 4.184 j) (round off answer to 2 decimal place Explain one of the benefits of a federalist structure which of the following is not a benefit of stretching?a. increased speedb. better range of motion c. perform daily activities more effectively d. enhanced flexibility Sin A /1-cosA= cot1/2 PLEASE HURRY IM TIMED As the first president of the United States of America, George Washington had a group of men helping him out. This group was made up of the Secretary of State,Secretary of Treasury, Secretary of War, and the Attorney General. What was this group called?The CabinetThe Justice LeagueThe Continental CongressThe Common Defense Which of the following is not included in an expository essay?conclusionopinionsintroductionbody paragraphs HEEEEEEELLLLLLPPPPPPP MEEEEEBecause Australia is one of the freest economies in the world, which of the following outcomes is experienced by its citizens?A.few restrictions on tradeB.heavy government regulation of businessC.difficulty in starting a new businessD.government ownership of all property Rashaad leans a 22-foot ladder against a wall so that it forms an angle of 65 with the ground. How high up the wall does the ladder reach? Round vour answer to the nearest hundredth of a foot if necessary. what idea was taken from the magna carta and included in the constitution? Gantt charts are effective for project scheduling if: a. Activities are highly interconnected b. Activity times are random c. The project contains simple activity sequencesd. All of the above In the great Gatsby Gatsby Daisy and Tom pursue the American dream identify one of the aforementioned characters dreams and why they wanted to consider with the characters day to pursue the dream and what character traits those actions reveal.PLEASE HELP WITH A THESIS STATEMENT FOR DAISY AS TO WHY HER AMERICAN DREAM IS TO LIVE A LIFE FULL OF WEALTH AND GLAMOUR!!ILL MARK BRAINLIEST!!! How important is it for a cell to maintain the same number of chromosomes? Explain. Consider the example below. Explain in your own words why the equation is wrong. (4^-2)^2=1/16Someone please help! Please answer in your own words The resting membrane potential of a neuron is at-70 mV. Which of the following event(s) will depolarize the cell? Ex Ex. and Ea are 60, -90, and -63 mV. Oa) Na+ channels open and K+ channels open b) N Los padres de Alvaro se van de viaje y le dejan una nota a su hijo con algunas cosas que tiene que hacer. Completa la nota con el presente del subjuntivo de los verbos entre parntesis. incorrectQuestion 1 with 7 blanksNo te olvides! Tambin l abreviaba el rezar y la mitad de la oracin noacababa, porque me tena mandado que en yndose el que la mandaba rezar, le tirase por el extremo del capuz. Yo as lo haca. Luego l tornaba a dar voces, diciendo:-Mandan rezar tal y tal oracin? -como suelen decir.Usaba poner juntos a si un jarrillo de vino, cuando comamos,y yo muy de presto le asa y daba un par de besos callados ytornbale a su lugar. Mas durme poco, que en los tragos conocala falta y, por reservar su vino a salvo, nunca despusdesamparaba el jarro, antes lo tena por el asa asido. Mas nohaba piedra imn que atrayese tanto como yo con una paja largade centeno que para aquelmenester tena hecha, la cual metindolaen la boca del jarro, chupando el vino, lo dejaba a buenas noches.Mas, como fuese el traidor tan astuto, pienso que me sinti, yde all en adelante cambi de propsito y colocaba su jarro entrelas piernas y tapbalo con la mano, y as beba seguro.